Achei uma solucao aqui :
http://mathoverflow.net/questions/16721/egz-theorem-erdos-ginzburg-ziv

Em 22 de outubro de 2012 09:02, terence thirteen <peterdirich...@gmail.com>
escreveu:

> Lembrei vagamente deste problema, mas acho que ele é mais complicado
> do que imaginamos.
>
> Lembro que num livro de Ross Honsberger, talvez o Math. Gems III, ele
> coloca uma demonstração para n sendo potência de 2, usando uma espécie
> de indução. E afirma que é verdadeiro no caso geral mas sem
> demonstrar.
>
> Vou ver se o pessoal do MathLinks pode dar uma luz...
>
> Em 19 de outubro de 2012 22:00, terence thirteen
> <peterdirich...@gmail.com> escreveu:
> > Em 19 de outubro de 2012 21:44, Gabriel Dalalio
> > <gabrieldala...@gmail.com> escreveu:
> >> Parece que realmente sempre existe, mas ainda estou em busca de uma
> prova ou
> >> alguém que saiba provar...
> >>
> >> E também quero obter um algoritmo para achar uma dessas subsequencias...
> >>
> >> Em 19 de outubro de 2012 16:50, Pedro Nascimento <pedromn...@gmail.com>
> >> escreveu:
> >>
> >>> *subconjuntos com a dada propriedade
> >>>
> >>> Em 19 de outubro de 2012 16:48, Pedro Nascimento <pedromn...@gmail.com
> >
> >>> escreveu:
> >>>
> >>>> Alguem conseguiu algo nesse problema? Parece uma boa conjectura,
> cheguei
> >>>> a simular so pra ver o comportamento, a quantidade de subconjuntos em
> um
> >>>> caso aleatorio eh beem grande e cresce rapido com n.
> >>>>
> >>>> Em 15 de outubro de 2012 21:53, Gabriel Dalalio
> >>>> <gabrieldala...@gmail.com> escreveu:
> >>>>
> >>>>> Eu pensei em casa dos pombos mas não consegui muita coisa, arranjar
> um
> >>>>> subconjunto qualquer que a soma seja divisível por n é facil, o
> problema é
> >>>>> ter exatamente n elementos.
> >>>>>
> >>>>> Em 15 de outubro de 2012 20:24, terence thirteen
> >>>>> <peterdirich...@gmail.com> escreveu:
> >>>>>
> >>>>>> Em 15 de outubro de 2012 18:49, Gabriel Dalalio
> >>>>>> <gabrieldala...@gmail.com> escreveu:
> >>>>>> > Eae galera, beleza?
> >>>>>> >
> >>>>>> > Eu estou pensando na seguinte situação:
> >>>>>> >
> >>>>>> > É dado um conjunto de inteiros de 2n elementos.
> >>>>>> > Sempre existe um subconjunto de n elementos tal que sua soma é
> >>>>>> > divisível por
> >>>>>> > n?
> >>>>>>
> >>>>>> Talvez um casa-dos-pombos?
> >
> > Pensei em algo neste estilo: para cada n-conjunto do conjunto de 2n
> > elementos, pareie com seu complementar. A ideia seria provar que a
> > soma 0 módulo n tem que surgir obrigatoriamente em algum momento.
> >
> > Vou pensar um tanto mais nisso aí...
> >
> >>>>>>
> >>>>>> > E será que sempre existem pelo menos dois subconjuntos diferentes
> com
> >>>>>> > essa
> >>>>>> > propriedade?
> >>>>>> >
> >>>>>> > Eu só consegui achar exemplos com no mínimo dois subconjuntos
> >>>>>> > possíveis,
> >>>>>> > por exemplo, um conjunto formado por n elementos 0 e n elementos
> 1.
> >>>>>> >
> >>>>>> > Alguém sabe responder essas perguntas?
> >>>>>> >
> >>>>>> > Obrigado,
> >>>>>> > Gabriel Dalalio
> >>>>>>
> >>>>>>
> >>>>>>
> >>>>>> --
> >>>>>> /**************************************/
> >>>>>> 神が祝福
> >>>>>>
> >>>>>> Torres
> >>>>>>
> >>>>>>
> >>>>>>
> =========================================================================
> >>>>>> Instru�ões para entrar na lista, sair da lista e usar a lista em
> >>>>>> http://www.mat.puc-rio.br/~obmlistas/obm-l.html
> >>>>>>
> >>>>>>
> =========================================================================
> >>>>>
> >>>>>
> >>>>
> >>>
> >>
> >
> >
> >
> > --
> > /**************************************/
> > 神が祝福
> >
> > Torres
>
>
>
> --
> /**************************************/
> 神が祝福
>
> Torres
>
> =========================================================================
> Instru�ões para entrar na lista, sair da lista e usar a lista em
> http://www.mat.puc-rio.br/~obmlistas/obm-l.html
> =========================================================================
>

-- 
Esta mensagem foi verificada pelo sistema de antiv�rus e
 acredita-se estar livre de perigo.

Responder a